Đến nội dung

viet9a14124869 nội dung

Có 855 mục bởi viet9a14124869 (Tìm giới hạn từ 30-04-2020)



Sắp theo                Sắp xếp  

#707144 $\frac{1}{a^2+2}+\frac{1}{b...

Đã gửi bởi viet9a14124869 on 27-04-2018 - 23:18 trong Bất đẳng thức và cực trị

 

Thú vị :D có thể làm trội lên nữa nhưng mình sẽ không nêu ra vì một vài lý do :)

$$ \frac{1}{a^2+2}+\frac{1}{c^2+2} \ge \frac{1}{(a+c)^2+2} +1/2 $$
$$ \Leftrightarrow ac(8-4ac-2a^2c^2-a^3c-ac^3) \ge 0 \text{ (Luôn đúng)} $$
@viet9a14124869: dồn về thế này đơn giản hơn chứ chia trường hợp kia có vẻ hơi "cực khổ" :)

 

Tui có nghĩ đến bổ đề này trước khi nghĩ đến bổ đề trên kia rồi , cơ mà quên mất $a+c\leq 1$ =)) .....




#707140 $\frac{1}{a^2+2}+\frac{1}{b...

Đã gửi bởi viet9a14124869 on 27-04-2018 - 21:40 trong Bất đẳng thức và cực trị

Cho các số không âm $a, b, c$ thỏa mãn $a+b+c=1$. Chứng minh rằng:

$\frac{1}{a^2+2}+\frac{1}{b^2+2}+\frac{1}{c^2+2}\geq \frac{4}{3}$

- Sáng tác- 

Lần đầu mình gặp dạng này , cũng hay phết , không biết Khoa Linh còn bài nào khác tương tự không ??

Lời giải :

Giả sử c = max {a; b; c}  

i. Nếu $c\leq \frac{1}{2} $ thì $VT\geq \frac{3}{c^2+2}\geq \frac{4}{3}$ 

ii. Nếu $c\geq \frac{1}{2}$ . Ta có một đánh giá sau : 

$$\frac{1}{a^2+2}+\frac{1}{b^2+2}\geq \frac{2}{(a+b)^2+2}$$ 

$$\Leftrightarrow (a^2+b^2)^2+2ab(a^2-ab+b^2)+2a^2+8ab+2b^2\geq 0$$ 

Điều này đúng vì a,b không âm .

Do đó kết hợp điều kiện a+b=1- c thì bất đẳng thức cần chứng minh tương đương với : 

$$\frac{1}{c^2+2}+\frac{2}{(c-1)^2+2}\geq \frac{4}{3}$$

$$\Leftrightarrow (c-1)(2c-1)(2c^2-c+3)\leq 0$$ 

Điều này luôn đúng với điều kiện ta xét ở trên .

Vậy bài toán được chứng minh , dấu bằng xảy ra khi (a;b;c)=(0;0;1) và các hoán vị của nó .




#706967 [TOPIC] ÔN THI PHƯƠNG TRÌNH THPT CHUYÊN 2018 - 2019

Đã gửi bởi viet9a14124869 on 25-04-2018 - 22:58 trong Tài liệu - Đề thi

Dưới đây là một bài toán mà mình nghĩ các bạn nên tiếp xúc với nó từ THCS ,một bài rất hay và có nhiều ứng dụng :D 

Bài toán số 9 : Giải phương trình nghiệm nguyên :

$$x^2+y^2+z^2=3xyz$$

 

P/S : Chứng minh rằng nếu (a;b;c) là một nghiệm của phương trình thỏa mãn a>b>c thì (3ac-b-a)(3ab-c-a)(3bc-2a)<0 




#706955 [TOPIC] ÔN THI BẤT ĐẲNG THỨC $\boxed{\text{THPT CHUYÊN}}$...

Đã gửi bởi viet9a14124869 on 25-04-2018 - 22:05 trong Tài liệu - Đề thi

 

Bài 91:

Cho 3 số thực dương a, b, c thỏa mãn abc=1. Chứng minh:

$$ \sqrt{\frac{a}{b+3}}+\sqrt{\frac{b}{c+3}}+\sqrt{\frac{c}{a+3}} \ge \frac{3}{2} \text{   (Vasile Cirtoaje)} $$

 

Bài này mình có nghĩ đến một cách làm sau :

Trước tiên ta có một bổ đề : Với a,b,c thực không âm , ta có $3(a^2b+b^2c+c^2a) \leq (a+b+c)(a^2+b^2+c^2)$ (*)

Chứng minh : Bất đẳng thức (*) tương đương với $a(a-b)^2+b(b-c)^2+c(c-a)^2\geq 0$ ( luôn đúng ) 

Quay lại bài toán , áp dụng bất đẳng thức Cauchy-Schwarzt

LHS=$\frac{a}{\sqrt{a(b+3)}}+\frac{b}{\sqrt{b(c+3)}}+\frac{c}{\sqrt{c(a+3)}}\geq \frac{(\sqrt{a}+\sqrt{b}+\sqrt{c})^2}{\sum \sqrt{a(b+3)}}\geq \frac{(\sqrt{a}+\sqrt{b}+\sqrt{c})^2}{\sqrt{(\sum \sqrt{a}).[\sum (b+3)\sqrt{a}]}}\geq \frac{((\sqrt{a}+\sqrt{b}+\sqrt{c})^2)}{\sqrt{(\sum \sqrt{a}).[3\sum \sqrt{a}+\frac{1}{3}(\sum \sqrt{a}).\sum a]}}=\frac{(\sum \sqrt{a})}{\sqrt{3+\frac{a+b+c}{3}}}$

Do đó ta cần chứng minh :$\frac{\sum \sqrt{a}}{\sqrt{3+\frac{a+b+c}{3}}}\geq \frac{3}{2}\Leftrightarrow a+b+c+8(\sqrt{ab}+\sqrt{bc}+\sqrt{ca})\geq 27$

Bất đẳng thức cuối đúng theo bất đẳng thức Cauchy với điều kiện abc=1 .

Dấu bằng xảy ra khi a=b=c=1 .

 

P/S 01: Theo mình thì kỹ năng trình bày cũng rất quan trọng , nên cũng mong các bạn đăng bài giải có thể chia sẻ , phô diễn nó cho các bạn khác học tập theo  ^_^ ... 

 

P/S 02 : Mình nhận ra bài này cũng có thể đổi biến rồi dùng bất đẳng thức Cauchy-Schwarzt ,cũng là một cách rất ngắn gọn và đẹp =)) ....




#705764 Đề luyện tập Olympic marathon VMF khối 10 lần 3 tuần 3 tháng 7 2017

Đã gửi bởi viet9a14124869 on 13-04-2018 - 18:56 trong Thi HSG cấp Tỉnh, Thành phố. Olympic 30-4. Đề thi và kiểm tra đội tuyển các cấp.

 

$ \boxed{\text{Bài 4:}} $
Cho tam giác $ABC$ với tâm nội tiếp là $I$, ngoại tiếp là $O $. Gọi $A' = AI \cap BC$. Đường tròn $(I)$ tiếp xúc với $BC,CA, AB$ lần lượt tại $D,E$ và $F$. 
a)Gọi $P = (AA'D) \cap (ABC) $. Tương tự như vậy ta có $Q,R$. Chứng minh rằng: $DP, EQ , FR$ đồng quy trên $OI$
b)  Gọi $X= IP \cap EF$. Tương tự ta dựng $Y,Z$. Chứng minh rằng $AX, BY$ và $CZ$ đồng quy tại một điểm thuộc $OI$.
 

Lời giải của mình : 

Câu a, Giả sử PD cắt OI và đường tròn ngoại tiếp tam giác ABC tại M , A'' . Ta có : 

$\widehat{PAA'}=\widehat{PDB}=\frac{1}{2}\widehat{POB}+\frac{1}{2}\widehat{A"OC}=\widehat{PAB}+\widehat{CAA"}\Rightarrow \widehat{CAA"}=\widehat{BAA'}=\frac{1}{2}\widehat{BAC}\Rightarrow \overline{A,A',A"}$

Gọi r,R là bán kính đường tròn nội tiếp và ngoại tiếp tam giác ABC . Do A" là điểm chính giữa cung BC( không chứa A ) nên OA"  song song với ID.

Theo định lý Thales dạng đại số : $\frac{\overline{MI}}{\overline{MO}}=\frac{\overline{ID}}{\overline{OA"}}=\frac{r}{R}$

Suy ra M cố định . Làm tương tự như trên ta chứng minh được DP,EQ,FR đồng quy tại điểm M nằm trên OI ( đpcm) . 




#704752 $\frac{a^{2}+b^{2}+c^{2}}...

Đã gửi bởi viet9a14124869 on 02-04-2018 - 20:15 trong Bất đẳng thức và cực trị

Chỗ này ngược dấu rồi bạn ơi!

Ừ ha , chết thật ẩu quá :D , để mình suy nghĩ lại ...




#704749 $\frac{a^{2}+b^{2}+c^{2}}...

Đã gửi bởi viet9a14124869 on 02-04-2018 - 19:58 trong Bất đẳng thức và cực trị

Cho $0\leq a,b,c\leq 2$ và $a+b+c=3$. Tìm GTNN của A = $\frac{a^{2}+b^{2}+c^{2}}{a^{3}+b^{3}+c^{3}}$

Bằng vài phép thử ta thấy dấu bằng bài toán đạt tại a=0,b=1,c=2 và các hoán vị, khi đó min A = $\frac{5}{9}$

+) Xét nếu a=b=c thì A=1 

+) Nếu a,b,c không đồng thời bằng nhau , ta có : 

$$\frac{a^2+b^2+c^2}{a^3+b^3+c^3}\geq \frac{a^2+b^2+c^2}{a^3+b^3+c^3-3abc}=\frac{a^2+b^2+c^2}{3(a^2+b^2+c^2-ab-bc-ca}$$

Do đó ta cần chứng minh $9(a^2+b^2+c^2)>=15(a^2+b^2+c^2-ab-bc-ca)\Leftrightarrow a^2+b^2+c^2\leq 5$  ( do a+b+c=3)

Giả sử a=max{a;b;c} thì $a\geq 1$ , từ đó :

$$a^2+b^2+c^2\leq a^2+(b+c)^2=a^2+(3-a)^2=2(a-1)(a-2)+5\leq 5$$ ( đpcm) .

 

Vậy bài toán được giải quyết ,min A = $\frac{5}{9}$.




#704746 Giải phương trình: $4^x+12.2^x+32=0$

Đã gửi bởi viet9a14124869 on 02-04-2018 - 19:39 trong Phương trình - hệ phương trình - bất phương trình

Đặt $t=2^{x}\implies t^2+12t+32=0\iff (t+4)(t+8)=0\iff t=-4;t=-8\iff x=-2;x=-3$

Chắc chắn chỗ này sai rồi nhé . 

 

Giải phương trình: $4^x+12.2^x+32=0$

Bài này mình nghĩ chỉ cần 1 đánh giá là Vế trái > 0 với mọi x là có thể kết luận vô nghiệm rồi  %%- .




#704369 tam giác $ABC$, $A(3,3),I(2,1)$ là tâm đường tròn ngoại t...

Đã gửi bởi viet9a14124869 on 26-03-2018 - 21:33 trong Phương pháp tọa độ trong mặt phẳng

tam giác $ABC$, $A(3,3),I(2,1)$ là tâm đường tròn ngoại tiếp tam giác. phương trình phân giác trong $A$ là $x-y=0$, $BC=\frac{8}{\sqrt{5}}$.tìm tọa độ $B,C$

Đơn giản : Theo đề ra  (I;IA) : $(x-2)^2+(y-1)^2=5$

Gọi giao của phân giác trong góc A với đường tròn (I;IA)  là X, suy ra X(0,0) 

Ta có IX vuông góc BC mà $\overrightarrow{XI}= (2;1)\Rightarrow BC : 2x+y+c =0$ ( c là tham số ) 

Gọi giao của IX với BC là N . Theo định lý Pythagores thì $IN=\sqrt{IB^2-\frac{BC^2}{4}}= \frac{3}{\sqrt{5}}$ 

Do đó $d_{I/BC}=\frac{3}{\sqrt{5}}\Leftrightarrow |5+c| = 3 \Leftrightarrow c \in \left \{ -8;-2 \right \}$

Viết phương trình đường thẳng BC rồi tìm tọa độ giao điểm .......

Kết luận : ........




#699598 Bất đẳng thức

Đã gửi bởi viet9a14124869 on 03-01-2018 - 21:52 trong Bất đẳng thức và cực trị

Cho các só thực dương a,b,c. Chứng minh rằng:

$\sqrt{(a^{2}b+b^{2}c+c^{2}a)(ab^{2}+bc^{2}+ca^{2})}\geq abc+\sqrt[3]{(a^{3}+abc)(b^{3}+abc)(c^{3}+abc)}$

Lâu không đăng bài :3

Dùng bất đẳng thức Bunyakowski , ta có :          LHS =$\sqrt{(a^2b+b^2c+c^2a)(c^2b+b^2a+ca^2)}\geq abc+(b^2+ac)\sqrt{ac}$

Làm tương tự và áp dụng bất đẳng thức AM-GM 

 $3LHS\geq3abc+(a^2+bc)\sqrt{bc}+(b^2+ca)\sqrt{ca}+(c^2+ab)\sqrt{ab}\geq 3abc+ 3 \sqrt[3]{abc(a^2+bc)(b^2+ca)(c^2+ab)}=3RHS $  

 

Vậy ta có điều phải chứng minh . Dấu bằng xảy ra khi a=b=c . 




#698257 TOPIC thảo luận, trao đổi toán thi học sinh giỏi khối 10,11 .

Đã gửi bởi viet9a14124869 on 14-12-2017 - 20:14 trong Chuyên đề toán THPT

Khởi động lại topic nhé mọi người :) 

Bài toán số 30 (sưu tầm ) : Cho điểm P cố định nằm trong đường tròn (O;R ) và hai điểm A,B chạy trên đường tròn đó sao cho $\widehat{APB}=90^o$ . Gọi M là trung điêm của dây AB và H là hình chiếu của P xuống AB . Chứng minh M,H luôn cùng thuộc một đường tròn cố định . 

Bài toán số 31 (sưu tầm ) : Cho tam giác ABC , Tìm quỹ tích điểm M thỏa mãn $MA^2+2MB^2-3MC^2=k$ ( k là số thực tùy ý ) 

 

P/S : Hai bài toán trên mình lấy trong mục tích vô hướng của các vector , mọi người tham gia giải nhé ... 

 

_______________________________________________________________________________________________

 

Mình sẽ tiếp thêm 1 bài đại số : 

Bài toán số 32 (sưu tầm ) : Cho hàm số $y=\left | \sqrt[4]{11+4x-x^4}-2m^2+5m-1 \right |$ . Tìm m để giá trị lớn nhất của hàm số là nhỏ nhất . 

 

                                            




#697845 2015-Junior Balkan Team Selection Test-Romania

Đã gửi bởi viet9a14124869 on 05-12-2017 - 22:39 trong Bất đẳng thức và cực trị

Day 2 : 

Problem 2 : Let a,b,c >0 such that $a\geq bc^2 $ ,$ b\geq ca^2$ ,$c\geq ab^2$ .Find the maximum of the expression :  

                                  $ S = abc(a-bc^2)(b-ca^2)(c-ab^2) $ 

Day 4 : 

Problem 2 : Solve in $\mathbb{N}^*$ equation : 

                                      $ 4^a.5^b-3^c.11^d = 1 $

 

P/S : Ai biết làm thì chỉ mình 2 câu này với ...




#697679 Cho $a,b,c >0$. Chứng minh rằng $\frac{a^3}{b}+\...

Đã gửi bởi viet9a14124869 on 03-12-2017 - 10:56 trong Bất đẳng thức và cực trị

Cho a,b,c dương. Chứng minh rằng $\frac{a^3}{b}+\frac{b^3}{c}+\frac{c^3}{a}\geq a\sqrt{ac}+b\sqrt{ba}+c\sqrt{cb}$

Ta có 1 phép dùng AM-GM rất đơn giản : $\frac{a^3}{b}+bc\geq 2a\sqrt{ac}$ 

Tương tự rồi cộng các bất đẳng thức cùng chiều , ta có 

$\frac{a^3}{b}+\frac{b^3}{c}+\frac{c^3}{a}+ab+bc+ca\geq 2a\sqrt{ac}+2b\sqrt{ba}+2c\sqrt{cb}$ (1)

Mặt khác ta lại có 1 đẳng thức khác hoàn toàn chứng mình được bằng AM-GM :

 $\frac{a^3}{b}+\frac{b^3}{c}+\frac{c^3}{a}\geq 2a^2+2b^2+2c^2-ab-bc-ca\geq ab+bc+ca$ (2) 

Từ (1) và (2) ta có đpcm . Dấu "=" xảy ra khi a=b=c . 




#697645 $2^{x}-1=3^{y}$

Đã gửi bởi viet9a14124869 on 02-12-2017 - 20:30 trong Đại số

đây là cách làm của mình bạn xem có đúng ko nhé 

nếu x là số chẵn suy ra y là số lẻ xét x theo môdun 8 suy ra đc y

 

Được chứ cách nào cũng được mà :) .




#697643 $2^{x}-1=3^{y}$

Đã gửi bởi viet9a14124869 on 02-12-2017 - 20:20 trong Đại số

giải phương trình nghiệm nguyên $2^{x}-1=3^{y}$

Mấu chốt làm mấy bài kiểu này thường là phát hiện dạng của số mũ :v

Ta thấy nếu x lẻ thì x có dạng 2k +1 $3^y=2^x-1=2^{2k+1}-1=4^{k}.2-1\equiv 1$ ( mod 3 )  $\Leftrightarrow y=0\Leftrightarrow x=1$

Nếu x chẵn . Đặt x=2k thì pt trở thành $3^y=2^x-1=2^{2k}-1=(2^k-1)(2^k+1)\rightarrow \left\{\begin{matrix} 2^k-1=3^a & \\ 2^k+1=3^b & \end{matrix}\right.$ 

Trong đó a,b nguyên dương , a+b=y và $3^b-3^a=2\Leftrightarrow 3^a(3^{b-a}-1)=2\rightarrow \left\{\begin{matrix} 3^a=1 & \\ 3^{b-a}-1=2 & \end{matrix}\right.\rightarrow \left\{\begin{matrix} a=0 & \\ b=1 & \end{matrix}\right.$ 

Từ đó tính ra nghiệm $(x;y)\in \left \{ (1;0)(2;1) \right \}$




#697521 TOPIC thảo luận, trao đổi toán thi học sinh giỏi khối 10,11 .

Đã gửi bởi viet9a14124869 on 30-11-2017 - 21:21 trong Chuyên đề toán THPT

$4=\sum \frac{1}{a}\geq \frac{1}{\sqrt[4]{abcd}}\Rightarrow \sqrt[4]{abcd}\leq 1$

Suy ra $\frac{a+b+c+d}{\sqrt[4]{abcd}}+\frac{4}{1+\sqrt[4]{abcd}}\geq 6$

P/S: Liệu chỉ thế này hay mình đã sai :))

Sai rồi , đoạn bôi đỏ bạn làm sai rồi nhé ....xem lại đi . 

 

P/S: Các trang trước  còn nhiều bài hay mà mình cũng không muốn bỏ , bạn nào giải được thì đăng lên cho mọi người xem với nhé .... :D  




#697194 TOPIC thảo luận, trao đổi toán thi học sinh giỏi khối 10,11 .

Đã gửi bởi viet9a14124869 on 26-11-2017 - 07:10 trong Chuyên đề toán THPT

Xin lỗi các mem vì thời gian qua mình bận 1 vài việc nên thỉnh thoảng có lên diễn đàn cũng không giữ lửa cho topic được :)) 

Bài 18 [Sưu tầm]: Cho $0<a<b<c$ thỏa mãn $a+b+c=6$ và $ab+bc+ca=9$. Chứng minh rằng: $0<a<1<b<3<c<4$

 

Bài này mình thấy nó khá giống bài British MO trong sách , mình sẽ trình bày theo cách trong sách này , ai có lời giải mới đẹp hơn thì up sớm để mọi người tham khảo nhé :)  có nhiều bài còn dư mong mọi người làm nốt để mình đăng thêm bài mới :)

Đặt p=abc ,xét hàm f(x) =(x-a)(x-b)(x-c) =$x^3-6x^2+9x-p$ 

                                f'(x) = $3x^2-12x+9=3(x-1)(x-3)$ 

Vậy f'(x) có hai nghiệm là x=1 và x=3 . Do f(x) có 3 nghiệm a<b<c nên 1<b<3 và f(1).f(3) <0 

Mặt khác f(1) =4-p  , f(3) =-p  , f(0) =-p  , f(4) =4-p 

Do đó 0<p<4 , suy ra f(0) <0 và f(4) >0 . Vậy 0<a<1<b<3<c<4 . 

 

P/S : Trong hôm nay nếu chưa ai có lời giải cho những bài còn lại thì mong các thành viên đã đề xuất bài toán đó đăng lời giải để mọi người tham khảo nhé ....... :D




#696552 TOPIC thảo luận, trao đổi toán thi học sinh giỏi khối 10,11 .

Đã gửi bởi viet9a14124869 on 13-11-2017 - 20:24 trong Chuyên đề toán THPT

Bài toán số 8(sưu tầm)

Cho A;B là các góc nhọn thỏa mãn $sinB=2005cos(A+B).sinA$

Tìm max tanB.

P/s mong mọi người ủng hộ topic

Lời giải :

Do A,B đều là góc nhọn nên ta có : $\left\{\begin{matrix} cosB\neq 0 & \\ sinA \in [0 ;1 ) & \end{matrix}\right.$

Theo đề ra ta có : sinB = 2005.sinA.cos(A+B) =2005.sinA . ( cosA.cosB - sinA.sinB ) 

 $\Leftrightarrow \frac{sinB}{cosB}=2005.sinA.(cosA-sinA.\frac{sinB}{cosB})$

$\Leftrightarrow tanB = 2005. sin A .(cosA-sinA.tanB)\Leftrightarrow tanB = \frac{2005.sinA.cosA}{2005.sin^2A+1}=\frac{2005.sinA.\sqrt{1-sin^2A}}{2005.sin^2A+1}$

Đến đây ta đặt n = sinA , có thể dễ dàng tìm max tanB bằng cách khảo sát hàm số với n $\in$ (0; 1) hoặc xử lý bằng cách dùng bất đẳng thức AM-GM :

tanB = $\frac{2005n\sqrt{1-n^2}}{2005n^2+1}=\frac{2005n\sqrt{1-n^2}}{2006n^2+(1-n^2)}\leq \frac{2005n\sqrt{1-n^2}}{2\sqrt{2006n^2(1-n^2)}}=\frac{2005}{2\sqrt{2006}}$

Vậy max tanB =$\frac{2005}{2\sqrt{2006}}$ , dấu "=" xảy ra khi sinA = $\frac{1}{\sqrt{2007}}$ . ^_^




#696470 TOPIC thảo luận, trao đổi toán thi học sinh giỏi khối 10,11 .

Đã gửi bởi viet9a14124869 on 12-11-2017 - 18:26 trong Chuyên đề toán THPT

Đăng hình có bị ném gạch không :D

Không nên spam như thế này bạn nhé :)) nếu có những câu hỏi khác tương tự vậy mong các bạn hãy trao đổi với mình qua tin nhắn

 

Mình có các bài như sau: 1,Tìm nghiệm nguyên của phương trình:$p^{2}-pq-q^{3}=1$

                                          2, Tìm các giá trị nguyên $(a,b)$ thỏa mãn: $a^{2}+b \vdots b^{2}-a;b^{2}+a \vdots a^{2}-b$

 

Tiếp lửa cho topic:

Bài 1: (Sưu tầm) Tìm tất cả các hàm số  $f:\mathbb{R}\rightarrow \mathbb{R}$ thỏa mãn: $f\left ( yf(x+y)+f(x) \right )=4x+2yf(x+y)$

Bài 2: (Sưu tầm) Giả sử rằng $f:\mathbb{R}\rightarrow \mathbb{R}$ là hàm số thỏa mãn điều kiện: $f(x^2-5x+1)+5f(x^2+x-5)=x^2-9, \forall x\in \mathbb{R}$. Hãy tìm $f(2009)$

Hai bạn nên đánh số thứ tự bài như các bạn khác đã làm ở trên , để cho dễ tìm dễ xem :3 mong hai bạn chú ý




#696154 TOPIC thảo luận, trao đổi toán thi học sinh giỏi khối 10,11 .

Đã gửi bởi viet9a14124869 on 06-11-2017 - 19:59 trong Chuyên đề toán THPT

Xin đề xuất 3 bài toán mới :

Bài toán số 3 ( sưu tầm ) : Cho phương trình $x^{12}+1-4x^4\sqrt{x^n-1}=0$ . Tìm số n nguyên dương bé nhất để phương trình có nghiệm.
Bài toán số 4 ( sưu tầm ) : Giải phương trình lượng giác :

a , Giải phương trình lượng giác : $cos^{3}(x)+sin (x)-3sin^{2}(x).cos(x)=0$

b ,   Tìm giá trị lớn nhất của biểu thức $P = (1+tan\frac{A}{4})(1+tan\frac{B}{4})(1+tan\frac{C}{4})$

                   ( Trong đó A,B,C là 3 góc của 1 tam giác )

Bài toán số 5 ( sưu tầm) : Tìm giới hạn : $\lim_{x\rightarrow 0}\frac{\sqrt{x^2-x+1}+\sqrt[4]{x^4-x+1}-2\sqrt[3]{x^3-x+1}}{x}$

 

 

P/S : Câu 4b mình có sửa lại đề cho phù hợp hơn rồi  , các thánh vô chém đi nào :)




#696077 TOPIC thảo luận, trao đổi toán thi học sinh giỏi khối 10,11 .

Đã gửi bởi viet9a14124869 on 04-11-2017 - 21:33 trong Chuyên đề toán THPT

 

Bài toán số 2 ( HSG 11 Quảng Bình 2016-2017 ) : Cho x,y,z >0 . Chứng minh :

$A=\frac{1+x^2}{1+4\sqrt{1+y^3}+3z^2}+\frac{1+y^2}{1+4\sqrt{1+z^3}+3x^2}+\frac{1+z^2}{1+4\sqrt{1+x^3}+3y^2}\geq \frac{3}{5}$ .

 

P/S : Mong các mem mà mod tích cực thảo luận và quan tâm đến topic nhiều hơn  .

Để ý một chút ta sẽ thấy dấu bằng của bài toán đạt tại 3 biến bằng nhau và bằng 2

Do đó áp dụng bất đẳng thức Cauchy ta có đánh giá sau :

$\frac{1+x^2}{1+4\sqrt{1+y^3}+3z^2}=\frac{1+x^2}{1+4\sqrt{(1+y)(1-y+y^2)}+3z^2}\geq \frac{1+x^2}{1+4.\frac{1+y+1-y+y^2}{2}+3z^2}=\frac{x^2+1}{3z^2+2y^2+5}$

Làm tương tự với 2 đẳng thức còn lại , để cho dễ nhìn hơn ta sẽ đổi biến $a=x^2+1 ,b=y^2+1 , c=z^2+1$ .Khi đó bài toán quay về chứng minh :

         A $\geq$ $\frac{a}{2b+3c}+\frac{b}{2c+3a}+\frac{c}{2a+3b}\geq \frac{3}{5}$

Thật vậy ta áp dụng bất đẳng thức Schwarz dạng Engel :

$A=\frac{a^2}{2ab+3ca}+\frac{b^2}{2bc+3ba}+\frac{c^2}{2ca+3cb}\geq \frac{(a+b+c)^2}{5(ab+bc+ca)}\geq \frac{3}{5}$

$\Leftrightarrow a^2+b^2+c^2\geq ab+bc+ca\Leftrightarrow (a-b)^2+(b-c)^2+(c-a)^2\geq 0$ ( Đúng )

Vậy bài toán được chứng minh . :biggrin:

 

P/S : Khuyến khích mọi người giải một bài toán bằng nhiều cách nha :)




#696074 TOPIC thảo luận, trao đổi toán thi học sinh giỏi khối 10,11 .

Đã gửi bởi viet9a14124869 on 04-11-2017 - 21:15 trong Chuyên đề toán THPT

Chào các bạn :) , hiện tại trên diễn đàn mình vẫn chưa có topic ôn thi học sinh giỏi lớp 10 , lớp 11 . Sau khi bàn bạc mình cùng với 2 bạn cristianoronaldoDinhXuanHung CQB đã quyết định lập một topic để các bạn đam mê có thêm các phương pháp ôn thi ,giải toán , tạo thêm 1 sân chơi cho các bạn đang chuẩn bị bước vào kỳ thi học sinh giỏi năm nay ...

Mở đầu topic này mình sẽ đề cập đến hai bài toán khá hay :

 

Bài toán số 1 (Sưu tầm ) : Cho số $n\in \mathbb{Z}^+$ và tập $S = \left \{ n^2+1;n^2+2;...;(n+1)^2-1 \right \}$ . Chứng minh rằng không thể tồn tại 4 phần tử phân biệt thuộc S mà tích hai phần tử này bằng tích hai phần tử kia .

 

Bài toán số 2 ( HSG 11 Quảng Bình 2016-2017 ) : Cho x,y,z >0 . Chứng minh :

$A=\frac{1+x^2}{1+4\sqrt{1+y^3}+3z^2}+\frac{1+y^2}{1+4\sqrt{1+z^3}+3x^2}+\frac{1+z^2}{1+4\sqrt{1+x^3}+3y^2}\geq \frac{3}{5}$ .

 

P/S : Mong các mem mà mod tích cực thảo luận và quan tâm đến topic nhiều hơn  .




#696061 Cho $a,b,c>0$ và $a+b+c$=3. CMR $\sum...

Đã gửi bởi viet9a14124869 on 04-11-2017 - 18:13 trong Bất đẳng thức và cực trị

Cho a, b, c> 0 thoả a+b+c = 3 CMR: 

Bài này đùng là làm theo hướng dùng bất đẳng thức Schwarz của Hùng , nhưng đoạn cuối đáng ra nên làm như vầy

Đặt biểu thức trên là A

Ta có $A=\frac{a}{a+2bc}+\frac{b}{b+2ca}+\frac{c}{c+2ab}\geq \frac{(a+b+c)^2}{a^2+b^2+c^2+6abc}\geq 1\Leftrightarrow ab+bc+ca\geq 3abc$

Đẳng thức này đùng vì theo bất đẳng thức AM-GM thì ta có :

$3=a+b+c\geq 3\sqrt[3]{abc}\rightarrow 1\geq abc\rightarrow ab+bc+ca\geq 3\sqrt[3]{a^2b^2c^2}\geq 3abc$

Vậy ta có đpcm ...dấu bằng xảy ra khi a=b=c=1 .

Một cách khác là ta có thể chứng minh bằng kỹ thuật đánh giá phủ định của phủ định hay còn gọi là AM-GM ngược dấu :3




#695899 $\sum \frac{a^3}{(2a^2+b^2)(2a^2+c^2)}...

Đã gửi bởi viet9a14124869 on 31-10-2017 - 21:05 trong Bất đẳng thức và cực trị

Cho a,b,c >0.Chứng minh: $\sum \frac{a^3}{(2a^2+b^2)(2a^2+c^2)}\leq \frac{1}{a+b+c}$

Bài này nhìn quen quen :v

Dùng một đánh giá dựa theo BĐT Bunyakovsky :

$\frac{a^3}{(2a^2+b^2)(2a^2+c^2)}=\frac{a^3}{(a^2+a^2+b^2)(a^2+c^2+a^2)}\leq \frac{a^3}{(a^2+ab+ac)^2}=\frac{a}{(a+b+c)^2}$

Làm tương tự rồi cộng vế theo vế là ra

Dấu bằng xảy ra khi a=b=c :D




#695865 Tìm cách giải mới

Đã gửi bởi viet9a14124869 on 30-10-2017 - 22:55 trong Phương trình - hệ phương trình - bất phương trình

Đáp án của mình là $m=2^{(n-1)*(n+1)/8}$  với $n\geq 1$

Bạn nào có cách giải hoặc đáp án khác không?

Ở trên mình nhầm tí đã sửa lại ,  bạn xem lại hộ mình với :3